Exam 6

अब Quizwiz के साथ अपने होमवर्क और परीक्षाओं को एस करें!

List the pathophysiologic mechanisms that occur in asthma in order.

Answer: 1. Exposure to trigger 2. Blood vessels dilate 3. Bronchioles constrict 4. Lung dysfunction

The nurse anticipates a diagnosis of sinusitis for a patient who reports a headache, nasal congestion, and fever for the past 3 days as indicated by which assessment findings? Select all that apply. A. Nosebleed B. Swollen turbinates C. Edematous mucosa D. Clear nasal discharge E. Tenderness over the sinuses

Answers: B. Swollen turbinates C. Edematous mucosa E. Tenderness over the sinuses Rationale: Clinical findings that indicate sinusitis include swollen turbinates, edematous mucosa, tenderness over the involved sinuses, and enlarged turbinates. The turbinates may enlarge due to congestion. The inflammation results in increased blood supply to the affected area, which leads to edematous mucosa.Due to the inflammation, there may be tenderness over the involved frontal and/or maxillary sinuses. Clear nasal discharge is not a sign of sinusitis. Nosebleed is not a manifestation of sinusitis. Patients with sinusitis usually have a purulent nasal discharge. p. 485

Place the physical manifestations of chronic obstructive pulmonary disease in the order in which they appear in the disease process.

Answer: 1. Productive cough every few days for a month 2. Decreased endurance during morning walks 3. Difficulty breathing on awakening in the morning 4. Altered chest expansion 5. Bluish skin discoloration Rationale: Patients with COPD usually develop a persistent cough first, then note mild activity intolerance; which progresses to significant dyspnea at rest. Hyperinflation of the lungs occurs as a result of air trapping, and progressive hypoxemia leads to polycythemia, which can cause a bluish-red discoloration of the patient's skin color.

A patient arrives at the hospital with a history of long-term exposure to caustic fumes. Assessment reveals a forced expiratory volume in 1 second/forced vital capacity (FEV1/FVC) ratio of 65% and a functional oxygen saturation of 88%. The patient smokes 1 pack of cigarettes per day and reports a recent increase in sputum production and a change in color from clear to green. In which order should the nurse initiate the collaborative care actions?

Answer: 1. Provide supplemental oxygen. 2. Obtain a sputum culture. 3. Administer intravenous corticosteroids. 4. Teach diaphragmatic breathing. Rationale: The nurse should first apply oxygen to treat tissue hypoxemia. After applying oxygen, the nurse should instruct the patient how to effectively breathe from the diaphragm. Next corticosteroids would be administered intravenously to treat inflammation. Next, the nurse should obtain a sputum culture to determine whether an infection is causing the change in sputum color.

Place the pathological process of pneumonia in order.

Answer: 1. Release of cytokines, histamines, and prostaglandins 2. Obstruction of the airway with mucus 3. Impaired CO2 and O2 exchange 4. Mucus and debris removed by alveolar macrophages 5. Lung tissue functioning normally

A health care provider advises against prescribing an antibiotic to a patient with a severe cold, sore throat, and runny nose. The patient states, "I really think that an antibiotic would make me feel better soon." How does the nurse respond? A. "An antibiotic is ineffective against a cold or flu." B. "Do you have any previously prescribed antibiotic at your home? If so, you can take that." C. "Your symptoms aren't severe enough to treat with antibiotics." D. "I will review a list of antibiotics; I will ask your health care provider to prescribe one of them."

Answer: A. "An antibiotic is ineffective against a cold or flu." Rationale: A cold, sore throat, and runny nose are symptoms of cold and flu (a viral infection), and antibiotics are ineffective in treating viral infections. The nurse should inform the patient about the use of antibiotics, not tell the patient to use the rest of a previously prescribed antibiotic from home. Telling the patient that the rationale for the health care provider's decision is that the symptoms aren't severe enough is not accurate. at the nurse should not ask the health care provider to prescribe antibiotics. p. 215

A patient is being treated for pneumonia. Which statements, by the patient, indicate that patient teaching was successful? A. "Every year I should get a flu vaccine." B. "I'll be able to resume my jogging routine in a week or two." C. "I can have one beer while taking the antibiotic." D. "I will use a warm mist humidifier while I sleep at night."

Answer: A. "Every year I should get a flu vaccine." Rationale: A yearly flu vaccine is recommended for patients with pneumonia.

A patient presents with symptoms of pneumonia. A sputum culture identifies the infectious agent as bacterial in nature and the patient is placed on IV antibiotics. Within 48 hours, the patient's vital signs return to normal, the patient has switched to oral antibiotics, is drinking fluids normally, and has had two formed bowel movements. The patient asks the nurse, "When may I go home?" Based on the patient's condition, what is the best response by the nurse? A. "You will likely be discharged soon." B. "That will depend on your blood work." C. "You will need to be observed for several more days." D. "You will beed to remain in the hospital to complete the antibiotic course."

Answer: A. "You will likely be discharged soon." Rationale: A patient who is stable, able to ingest oral substances, and is exhibiting normal bowel movements may be discharged with oral antibiotics.

Which patient conditions increase the risk of contracting pneumonia? Select all that apply. A. AIDS B. Coma C. Hyperthermia D. Pleural effusion E. Congenital heart defect

Answer: A. AIDS B. Coma E. Congenital heart defect Rationale: Suppression of the immune system can cause an overgrowth of bacteria, leading to an increased risk for pneumonia. Intubation would be implemented for a comatose patient. Both intubation and decreased consciousness interfere with the normal cough reflex, allowing bacteria to grow, and causing pneumonia. Heart disease, such as congenital heart defect, can suppress the immune system's ability to inhibit bacterial growth, leading to an increased risk for pneumonia.

A patient presents with productive cough with green sputum, a temperature of 102.5 degrees F, and a SpO2 of 88%. Additionally, the patient reports severe pleural chest pain. The patient is diagnosed with viral pneumonia. What is the appropriate collaborative care for this patient? Select all that apply. A. Acetaminophen B. Supplemental oxygen C. Limit caloric intake to 1800 per day D. IV antibiotics E. Frequent liquid intake

Answer: A. Acetaminophen B. Supplemental oxygen E. Frequent liquid intake Rationale: Acetaminophen- The care for viral pneumonia is primarily supportive, and includes providing antipyretics for febrile patients. Supplemental oxygen- The care for viral pneumonia is primarily supportive and includes administering oxygen therapy. A patient with a SpO2 of 88% is in need of oxygen therapy. Patients with pneumonia need to stay hydrated and if hydration is not possible orally then IV fluids should be started.

Which interprofessional action will the nurse take first for a patient who has hypoxemia caused by pneumonia? A. Administer oxygen. B. Give prescribed antibiotic. C. Assist the patient to cough. D. Offer oral fluids frequently.

Answer: A. Administer oxygen. Rationale: The initial action in a patient with hypoxemia is administration of supplemental oxygen to improve oxygen saturation to 90% or better. Pneumonia causes ventilation-perfusion (V/Q) mismatch, and other treatments are likely to be needed as well. Antibiotics are prescribed to treat bacterial pneumonia (after blood and sputum cultures are obtained). Coughing helps clear airways and decrease the V/Q mismatch by improving ventilation. Increased fluid intake will help thin respiratory secretions and make them easier to expel. p. 1590

A 70-year-old patient presents to the emergency department with symptoms that indicate pneumonia. Assessment findings include a new onset of confusion, a respiratory rate of 42 breaths/minute, a blood urea nitrogen (BUN) of 24 mg/dL, and a BP of 80/50 mm Hg. The nurse expects which treatment plan? A. Admit the patient to the intensive care unit. B. Admit the patient to a general medical-surgical unit. C. Discharge the patient and monitor for signs of worsening infection. D. Admit the patient for a 23-hour observation and monitor for pulmonary embolism.

Answer: A. Admit the patient to the intensive care unit. Rationale: According to the Expanded CURB-65 scale, which is used as a supplement to clinical judgment to determine the severity of pneumonia, the patient's score is a 5; placement in the intensive care unit is recommended. The patient receives 1 point for each criterion: confusion (compared to baseline); NUM greater than 20 mg/dL; respiratory rate greater than or equal to 30 breaths/min; systolic BP of less than 90 mm Hg; and age greater than or equal to 65 yrs. With severe pneumonia, the patient needs a higher level of care than general medical-surgical. Discharging the patient is unsafe. Pulmonary embolism does not manifest in this way, and assessing for it is not required in this case. p. 504

Which prescribed medications will the nurse anticipate administering for a patient with respiratory failure caused by pneumonia? Select all that apply. A. ALbuterol B. Furosemide C. Levofloxacin D. Nitroglycerin E. Methylprednisolone

Answer: A. Albuterol C. Levofloxacin E. Methylprednisolone Rationale: Bronchodilators such as albuterol, antibiotics such as levofloxacin, and corticosteroids such as methylprednisolone are used in respiratory failure secondary to pneumonia to open airways, treat infection and decrease airway inflammation. A diurectic such as furosemide would be used for respiratory failure caused by pulmonary congestion due to heart failure but would likely lead to dehydration in a patient with pneumonia. A vasodilator such as nitroglycerin can also be used to decrease pulmonary congestion caused by left-sided heart failure but would not be helpful in treating pneumonia. pp. 1595-1596

Which inhaler would the nurse be prepared to administer to the patient at the onset of an asthma attack? A. Albuterol B. Fluticasone C. Salmeterol D. Tiotropium

Answer: A. Albuterol Rationale: Albuterol is short-acting bronchodilator that should be given first when the patient experiences an asthma attack. Fluticasone is an inhaled corticosteroid (ICS) used to prevent asthma attacks and does not result in rapid bronchodilation. Salmeterol is a long-acting bronchodilator and will not work rapidly to relieve bronchospasm in an acute attack. Tiotropium is a long-acting anticholinergic bronchodilator that is recommended only for use in chronic obstructive pulmonary disease and would not work to rapidly improve breathing in an asthma attack. pp. 549-550

A patient with asthma is admitted with severe dyspnea and is unable to speak. The nurse finds that the patient looks drowsy and confused. Which other finding would the nurse expect? A. Arterial blood gas deterioration B. Loud and prominent wheezing C. Peak flow of more than 40% D. Tachycardia-associated respiratory arrest

Answer: A. Arterial blood gas deterioration Rationale: A patient with a severe asthma exacerbation is dyspneic at rest and has difficulty speaking. The patient may be unusually drowsy and confused as the arterial blood gas levels deteriorate. Upon auscultation, the nurse would hear no wheezing sound in the patient with life-threatening asthma. Peak flow in the patient reduces 25% of the personal best because of limited airflow in the airway. The patient with a reduction in the peak flow of air experiences bradycardia with respiratory arrest. p. 546

A patient is hospitalized with a diagnosis of pneumonia. When reviewing the patient's history, the nurse finds that the patient experienced a seizure with profuse vomiting four days prior to the hospital admission. Which type of pneumonia does the nurse suspect? A. Aspiration pneumonia B. Opportunistic pneumonia C. Hospital-associated pneumonia D. Community-acquired pneumonia

Answer: A. Aspiration pneumonia Rationale: Aspiration pneumonia results from the abnormal entry of material from the mouth or stomach into the trachea and lungs. Conditions that increase the risk for aspiration include decreased level of consciousness (e.g., seizure, anesthesia, head injury, stroke, alcohol intake), difficulty swallowing, and insertion of nasogastric (NG) tubes with or without enteral feeding. The aspirated material (food, water, vomitus, oropharyngeal secretions) triggers an inflammatory response. The history of the patient does not suggest any exposure to pneumonia in the community. The patient has never been in the hospital; therefore hospital-associated pneumonia is highly unlikely. The patient does not have a history of HIV, intake of immunosuppressive drugs, corticosteroids, or any disorders leading to immunosuppression; therefore opportunistic pneumonia did not occur in this patient. p. 505

For which patient condition would the nurse withhold the prescribed timolol (Timoptic; Istalol), two drops to each eye every 12 hours, and contact the prescriber? A. Asthma B. Urinary retention C. Cluster headaches D. Chronic constipation

Answer: A. Asthma Rationale: Timolol is a nonselective Beta-adrenergic blocking agent that can cause bronchoconstriction and bronchospasm. For this reason, it should not be used in patients with severe chronic obstructive pulmonary disease (COPD) or asthma. Timolol will not increase urinary retention; that commonly is seen with use of anticholinergics. It will not cause or worsen headaches or constipation. p. 371

Which finding indicates to the nurse that a patient's respiratory status is improving following an acute asthma exacerbation? A. Audible wheezing B. Pursed lip breathing C. Use of intercostal muscles D. Oxygen saturation of 89% of room air

Answer: A. Audible wheezing Rationale: The primary problem during an exacerbation of asthma is narrowing of the airways and subsequent diminished air exchange. As the airways begin to dilate, the wheezing gets louder because of better air exchange. Pursed lip breathing does not correlate with asthma improvement. The use of intercostal muscles and an oxygen saturation of 89% are evidence of continued asthma exacerbation. pp. 545-546

Which action will the nurse take to improve oxygenation in a patient with left-sided pneumonia? A. Augmented coughing or huff coughing B. Fluid restriction of less than 2000 mL daily C. Positioning the patient side-lying on the left side D. Insertion of a nasogastric tube for enteral feedings

Answer: A. Augmented coughing or huff coughing Rationale: Augmented coughing and huff coughing techniques may aid the patient in the mobilization of secretions. Since respiratory secretions are usually thick and viscous in pneumonia, fluid intake of at least 2000 to 3000 mL daily is encouraged. A patient with left-sided pneumonia would be positioned on the right side to maximize perfusion to the good lung and improve gas exchange. Nutrition is important in patients with pneumonia, but there is no indication that the patient requires a nasogastric tube for feeding. pp. 1594-1595

Which findings will the nurse expect when assessing a clinic patient with chronic obstructive pulmonary disease (COPD)? Select all that apply. A. Barrel chest on inspection B. Egophony on auscultation C. Dullness on percussion D. Wheezes on auscultation E. Increased tactile fremitus on palpation

Answer: A. Barrel chest on inspection D. Wheezes on auscultation Rationale: Because of chest expansion caused by air trapping in COPD, patients develop a barrel chest. Breath sounds decrease because of less air movement caused by air trapping, and wheezes are heard because of narrowing of airways caused by inflammation and increased mucus production. Egophony is heard with consolidation of lung tissue, such as that which occurs in patients with pneumonia or pleural effusion. Dullness on percussion would indicate a consolidation of fluid or tissue in the lungs, which is not consistent with COPD. Tactile fremitus is decreased in COPD because trapped air leads to less transmission of vibration from the airways to the chest surface. p. 563

Which action may increase pain for a patient with a history of a herniated lumbar disc and low back pain? A. Bending or lifting B. Application of warm, moist heat C. Sleeping in a side-lying postion D. Sitting in a fully extended recliner

Answer: A. Bending or lifting

A patient presents with bacterial pneumonia and a productive, blood-tinged sputum. The patient's viral signs are an oral temperature of 102.8 degrees F, a respiratory rate 28 respirations per minute, an oxygen saturation of 85%, a heart rate of 80 beats per minute, and. a blood pressure reading of 125/90. Which of the reported information indicates that the antibiotic therapy is effective? A. Clear breath sounds B. Shortness of breath with anxiety C. A repeat chest x-ray indicates fluid D. Fine crackles in the lung bases bilaterally

Answer: A. Clear breath sounds Rationale: Clear breath sounds indicate that the bacterial pneumonia is resolving.

A patient presents to the hospital with symptoms of pneumonia. The patient is treated with IV antibiotics for five days and remains afebrile for 24 hours. What is the next step in the collaborative care of this patient? A. Continue treatment B. Stop the antibiotic therapy C. Change the dosage of antibiotics D. Administer the patient oral antibiotics

Answer: A. Continue treatment Rationale: The total treatment time for patients with community-acquired pneumonia should be a minimum of five days, and the patient should be afebrile for 48 to 72 hours before stopping treatment. This patient would need to be afebrile for an additional 24 hours before treatment may be discontinued.

A patient with chronic back pain tells the nurse, "I'd like to try more natural methods to control my pain. I'd rather not take any medications." Which information should the nurse provide the patient? A. Details on a local acupuncture studio B. Contact information for a pain relief clinic C. Recommend over-the-counter (OTC) herbal remedies D. Emphasis on the importance of taking medications as prescribed.

Answer: A. Details on a local acupuncture studio Rationale: Complementary therapy would not include use of medications and can be effective in treating pain.

When caring for a patient with chronic obstructive pulmonary disease (COPD) who has increased b-type natriuretic peptide (BNP) levels, which treatment will the nurse anticipate administering? A. Diuretic B. Antibiotic C. Oral corticosteroid D. Phosphodiesterase inhibitor

Answer: A. Diuretic Rationale: An increased BNP level in.a patient with COPD indicates likely heart failure. Diuretic use will decrease fluid overload and lower BNP, as well as reduce symptoms such as dyspnea. Antibiotic therapy would be expected if the patient had symptoms of respiratory infection such as fever and purulent sputum. An oral corticosteroid might be prescribed for a COPD exacerbation with increased dyspnea, wheezes, and rhonchi. A phosphodiesterase inhibitor might be prescribed for a patient who experienced frequent COPD exacerbations. p. 564

A patient presents to the emergency department with difficulty breathing and tightness in the chest. The patient reports being confined to bed or the couch all day, unable to perform any activities. The patient's peak flow measures 67% of the patient's personal best, according to the electronic medical record. Which action should the nurse take first? A. Give albuterol B. Give inhaled fluticasone. C. Administer an intravenous (IV) fluid bolus. D. Administer oral albuterol.

Answer: A. Give albuterol Rationale: Albuterol is a short-acting Beta2-agonist and is administered, sometimes with an oral corticosteroid, to control a mild-to-moderate exacerbation of asthma.

A patient with a history of asthma presents with wheezing and dyspnea and reports tightness of the chest. On assessment, the patient appears agitated and is fidgeting. The patient's vital signs are: blood pressure 160/90 mm Hg, temperature 99.0 F, heart rate 95 beats/min, and respiratory rate 19 breaths/min. Which complication of asthma is this patient likely experiencing? A. Hypoxemia B. Heart failure C. Bacterial pneumonia D. Accessory muscle use

Answer: A. Hypoxemia Rationale: Hypoxemia can lead to agitation and irritability.

Which patient education is critical for Ms. Asher to prevent and/or prepare for future attacks? A. Identification of asthma triggers B. The importance proper nutrition C. Keeping asthma inhalers on hand at all times D. Proper method for use of the peak flow meter

Answer: A. Identification of asthma triggers Rationale: Ms. Asher should be encouraged to identify triggers to prevent future attacks.

The nurse is assessing a patient with asthma who has been diagnosed with severe and life-threatening exacerbation. Which findings would the nurse expect to see? A. Increased CO2 level B. Speaking in short sentences C. Increased pH level on an arterial blood gas (ABG) D. Peak expiratory flow rate (PERF) of 70% of personal best

Answer: A. Increased CO2 level Rationale: Early in exacerbation, the CO2 level is decreased, but it increases if the exacerbation is prolonged or severe. With an asthma exacerbation, patients are unable to speak in sentences and only speak a few words at a time before taking a breath. The pH level on an ABG decreases as the episode is prolonged but is increased early in the exacerbation. With a life-threatening asthma attack, the PERF is 40% of the patient's personal best. p. 547

A healthy 70-year-old patient asks the nurse, "Which types of vaccinations are recommended for people my age?" Which vaccinations should the nurse suggest? Select all that apply. A. Influenza B. Pneumonia C. Meningococcal D. Hemophilus influenzae type b (Hib) E. Measles, mumps, and rubella (MMR)

Answer: A. Influenza B. Pneumonia Rationale: Older people are also more susceptible to infections (e.g., influenza, pneumonia) from pathogens that they were more immunocompetent against earlier in life. Bacterial pneumonia is the leading cause of death from infections in older adults. The patient should receive the shingles (herpes zoster), Pneumovax, and influenza vaccine. Meningococcal, Hib, and MMR vaccinations do not apply to this patient. Meningococcal vaccination is recommended for adults at risk (e.g., adults with anatomic or functional asplenia or persistent complement component deficiencies). Adults born before 1957 generally are considered immune to measles and mumps. Hib vaccination is considered only for adults with selected conditions (e.g., sickle cell disease, leukemia, human immunodeficiency virus [HIV] infection, or for those who have anatomic or functional asplenia) if they have not been vaccinated previously. p. 193

Which procedure is the preferred surgical treatment for vertebral compression fractures? A. Kyphoplasty B. Spinal fusion C. Vertebroplasty D. Bone mineral density (BMD)

Answer: A. Kyphoplasty Rationale: Kyphoplasty is the preferred treatment for vertebral compression fractures and uses a ballon inflation to insert bone cement. Vertebroplasty provides bone cement to stabilize the area and reduce pain. BMD is a test to determine peak bone mass. Spinal fusion uses bone to make a bridge between vertebrae that are next to one another. p. 1495

The nurse would MOST likely expect to see vertebral discs break down in which region of the spine? A. Lumbar B. Sacral C. Thoracic D. Cervical

Answer: A. Lumbar Rationale: The lumbar area is the most common area for vertebral disc breakdown because it is the most used area of the spine.

Interprofessional care for the management of osteoarthritis (OA) may be utilized for which treatment goals? Select all that apply. A. Managing pain B. Curing osteoarthritis C. Prevention of disability D. Managing inflammation E. Improving joint function F. Reviewing diagnostic labs

Answer: A. Managing pain C. Prevention of disability D. Managing inflammation E. Improving joint function

The nurse is caring for a patient who reports sudden onset of dyspnea and chest tightness. Oxygen saturation is 79%. The nurse notes a peak flow of 42%. Nebulizer albuterol is administered without improvement. Oxygen saturation after administration of albuterol is 81% and the patient is becoming fatigued. Which action would the nurse take next? A. Notify the provider B. Document the findings C. Give repeat dose of albuterol D. Prepare patient for intubation

Answer: A. Notify the provider Rationale: The nurse would notify the provider if the patient's symptoms have not improved after treatment.

A patient asks the nurse about the causes of low back pain. Which statement correctly describes the most common cause of low back pain? A. Overactivity of the lower back muscles causes pain and soreness. B. Vertebral breakdown causes low back pain and radiculopathy. C. Degenerative disk disease is the most common cause of low back pain. D. Scoliosis is a common spinal disorder in adults that can lead to low back pain.

Answer: A. Overactivity of the lower back muscles causes pain and soreness. Rationale: Overactivity is the most common cause of low back pain. Muscles and ligament fibers can be overstretched or injured during overactivity.

A nurse is caring for an older adult patient and suspects the patient may have a herniated disc. Which signs/symptoms are indicative of a herniated disc? A. Pain B. Numbness C. Leg weakness D. Cold sensation E. Swelling of the arm

Answer: A. Pain B. Numbness C. Leg weakness Rationale: Radiculopathy is a symptom of a herniated disc; this includes pain, numbness, and leg weakness caused by the distressed disc.

Which intervention would the nurse implement to prevent development of ventilator-associated pneumonia (VAP) in the intensive care unit? A. Promote early exercise and mobility. B. Perform hand washing during suctioning. C. Drain collected ventilator tubing water toward the patient. D. Maintain the head of the bed at a minimum of 20 degrees.

Answer: A. Promote early exercise and mobility. Rationale: Guidelines for VAP prevention include early exercise and mobility, minimizing sedation, use of endotracheal tubes with ports for drainage, head of bed at a minimum of 30 to 45 degrees unless medically contraindicated, no routine tubing changes, and strict hand washing before and after suctioning. Draining a ventilator toward the patient places the patient at risk for aspiration and infection. p. 1559

In which position would the nurse place the patient for a laminectomy? A. Prone B. Supine C. Lateral D. Lithotomy

Answer: A. Prone Rationale: Many positions are used in surgery; the choice is based on the type of surgery to be performed. For a laminectomy, the patient would be placed Ibn the prone position because it gives easy access to the back. The supine position is suited for surgery involving the abdomen, heart, or breast. The lateral position is best for surgery that involves one side of the body or the other. The lithotomy position is used for some types of pelvic organ surgery. p. 318

When caring for a patient admitted with a new diagnosis of chronic obstructive pulmonary disease (COPD), which medication prescribed by the health care provider will the nurse question? A. Roflumilast B. Albuterol C. Ipratropium D. Salmeterol

Answer: A. Roflumilast Rationale: Roflumilast is used for patients who have frequent COPD exacerbations and would not usually be prescribed for a patient with a new COPD diagnosis. Albuterol is short-acting bronchodilator and would typically be prescribed to control dyspnea in a patient with newly diagnosed COPD. Ipratropium is another short-acting bronchodilator that would commonly be prescribed to help control dyspnea in patients with COPD. Salmeterol is a long-acting bronchodilator, which would be recommended to minimize dyspnea in a patient with newly diagnosed COPD. pp. 551-552, 566

A nurse is caring for a child with a history of asthma. The nurse anticipates that the provider will order which test to determine whether the child's asthma is related to allergies? A. Serum immunoglobulin E (IgE) B. Chest x-ray C. Measurement of arterial blood gases (ABGs) D. Sputum culture

Answer: A. Serum immunoglobulin E (IgE) Rationale: IgE and eosinophil counts can be used to determine if the child's asthma is due to an allergic tendency.

A patient with inability to breathe through his nose who is assessed to find swelling of the nasal mucosa blocking the opening of the ostia may be experiencing which condition? A. Sinusitis B. Epistaxis C. Influenza D. Acute viral rhinitis

Answer: A. Sinusitis Rationale: Inflammation or swelling of the mucosa that blocks the opening of the ostia in the sinuses results in sinusitis. Epistaxis is bleeding from the nose, and this condition arises after nasal surgery or during sinusitis, trauma, or upper respiratory tract infections. Influenza is a respiratory illness caused by influenza virus. Acute viral rhinitis is an upper respiratory tract infection caused by an attack of rhinovirus, coxsackie viruses, or adenoviruses. p. 485

Which diagnostic test for a patient with a chronic cough will be most useful in making a diagnosis of chronic obstructive pulmonary disease (COPD)? A. Spirometry B. Chest x-ray C. Complete blood count (CBC) D. CT

Answer: A. Spirometry Rationale: Although all the tests might be used on a patient with a chronic cough and suspected COPD, spirometry is the best diagnostic test to confirm COPD. A forced expiratory volume/forced vital capacity (FEV1/FVC) ration less than 70% (0.70) will confirm the COPD diagnosis. Chest x-rays may be used to diagnose COPD exacerbations or to assess for lung hyperinflation but are not used to confirm a COPD diagnosis. A CBC may be prescribed for a patient with COPD to diagnose polycythemia or anemia, but changes in the CBC are not diagnostic for COPD. CT scans may be used to assess for lung changes associated with COPD but are not used to make the COPD diagnosis. p. 565

Which patient action may aggravate the acute sinusitis symptoms? A. Use of phenylephrine for five days B. Taking hot showers at least twice daily C. Applying warm, damp towels around the nose D. Keeping their pillow at a 45-degree angle while sleeping

Answer: A. Use of phenylephrine for five days Rationale: Phenylephrine is a decongestant used to treat sinusitis. This medication should not be used for more than three days because it causes rebound congestion due to vasodilation of the nasals. Taking hot showers helps to promote drainage of secretions; therefore the patient should take hot showers twice daily. A patient with sinusitis will exhibit pain at the affected sinuses; applying warm, damp towels around the nose will help to relieve the pain. The nurse will instruct the patient to elevate the head while sleeping in order to drain the sinuses and reduce nasal congestion. p. 486

A patient recently diagnosed with chronic low back pain tells the clinic nurse that she has added spinning classes to her exercise routine of step aerobics and weight training. The patient asks the nurse for any suggestions about other workouts to add. Which response should the nurse give to the patient? A. Swimming is a safe low-impact workout to add to your routine." B. "You should discuss your routine with your health care provider." C. "Exercise is an important part of managing your pain. You are doing great." D. "I think you are doing fine right now and should maintain this exercise routine."

Answer: B. "You should discuss your routine with your health care provider."

Which of these is a clinical manifestation of early chronic obstructive pulmonary disease (COPD)? A. Dyspnea at rest B. A chronic, intermittent cough C. The presence of chest breathing D. Increased numbers of red blood cells

Answer: B. A chronic, intermittent cough Rationale: Clinical manifestations of COPD typically develop slowly. A chronic intermittent cough, which is often the first symptom to develop, may later be present every day as the disease progresses. Dyspnea initially occurs with exertion, and dyspnea at rest is a typical in the later stages of COPD. Use of chest muscles such as the intercostals for breathing occurs in the late stages of COPD. Polycythemia occurs in the later stages of COPD as hypoxemia develops and stimulates increased red blood cell production. p. 563

A patient with low back pain relates working as a cashier at a local grocery store. Which item would the nurse teach the patient to keep nearby in the workplace to best assist with managing pain? A. Ice packs B. A footstool C. A nonskid mat D. A chair with high back support

Answer: B. A footstool

Which patient has the greatest risk for developing pneumonia? A. A conscious patient sitting in a chair B. A patient with tube feeding lying flat C. A patient with a tracheostomy with the head of bed elevated to 30 degrees D. An unconscious patient who is turned every 2 hours

Answer: B. A patient with tube feeding lying flat Rationale: This patient has the greatest risk for developing pneumonia. The patient should have the head of bed elevated to prevent aspiration pneumonia.

Which patient is most likely experiencing an acute asthma attack? A. A patient with fever and cough B. A patient with wheezing and nasal flaring C. A patient with productive cough and chills D. A patient with no lung sounds on the left side

Answer: B. A patient with wheezing and nasal flaring Rationale: Wheezing and nasal flaring indicate lower airway obstruction and use of accessory muscles, which are signs of an acute asthma attack.

Which patient is at the highest risk for developing chronic obstructive pulmonary disease (COPD)? A. A young adult with asthma and cystic fibrosis (CF) B. A smoker who works as a coal miner whose mother had COPD. C. An ex-smoker who works as a farmer who currently has pneumonia D. A patient with left-sided heart failure and a history of type 2 diabetes mellitus

Answer: B. A smoker who works as a coal miner whose mother had COPD Rationale: Exposure to cigarette smoke and coal dust increases the risk for COPD.

Which patient action when using albuterol and beclomethasone inhalers indicates a need for further teaching? A. Rinsing the mouth following use of the inhalers B. Administering the beclomethasone before the albuterol C. Administering the albuterol before the beclomethasone D. Administering the beclomethasone on a set schedule to prevent an asthma attack

Answer: B. Administering the beclomethasone before the the albuterol Rationale: Albuterol, a Beta2-adrenerfic agonist medication, should be used first to dilate the airways before administration of the corticosteroid beclomethasone, Administering the beclomethasone on a set schedule to prevent an asthma attack and rinsing the mouth following use of the inhalers are correct actions. p. 553

A patient arrives in the emergency department with severe wheezing and dyspnea that started 10 minutes ago. Which medications should the nurse anticipate administering to this patient? Select all that apply. A. Nonsteroidal anti-inflammatory drugs (NSAIDs) B. Albuterol C. Fluticasone D. Oral penicillin E. Intravenous (IV) methylprednisolone

Answer: B. Albuterol E. Intravenous (IV) methylprednisolone Rationale: Albuterol is a short acting beta2-agonist and would be given to dilate the bronchioles in patients with acute asthma exacerbations. Methylprednisolone is a corticosteroid that may be used to treat inflammation in acute asthma exacerbations. IV corticosteroids are used for management of acute asthma exacerbations.

A nurse asks the patient to imagine tiny elastic rubber bands that constrict the airways opening up to facilitate the free flow of air through the airways. In which condition would this example of imagery be helpful? A. Cancer B. Asthma C. Depression D. Coronary heart disease

Answer: B. Asthma Rationale: Imagery such as tiny elastic rubber bands that help to open constricted airways is effective in patients with asthma. The patient may feel the airway getting cleared out. Patients with cancer may benefit from imagery that includes sharks gobbling up the cancer cells. Patients with coronary artery disease may benefit from imagery that involves water flowing freely through a wide, open river. Patients with depression may benefit from images that involve troubles and feelings of sadness attached to big colorful helium balloons that are floating off into a clear, blue sky. p. 84

Which lifestyle changes may help to prevent osteoarthritis? Select all that apply. A. Avoiding alcohol B. Avoiding smoking C. Avoiding heavy lifting D. Maintaining a healthy weight E. Promptly treating any joint injury F. Sliding objects rather than lifting them

Answer: B. Avoiding smoking D. Maintaining a healthy weight E. Promptly treating any joint injury Rationale: Maintaining a healthy weight, avoiding smoking, and promptly treating any joint injuries are all measures the patient can take to prevent osteoarthritis. p. 1504

A patient with a history of asthma presents to the emergency department with wheezing, chest pain, and shortness of breath. Which type of medication listed in the patient's medication profile may be responsible for the patient's symptoms? A. Short-acting nitrate B. Beta-adrenergic blocker C. Calcium channel blocker D. Angiotensin II receptor blocker

Answer: B. Beta-adrenergic blocker Rationale: The patient with a history of asthma should avoid Beta-adrenergic blockers because the side effects include wheezing from bronchospasm. Short-acting nitrates are the first-line treatment for a patient with angina and can be used safely by patients who have asthma. Calcium channel blockers are used in patients who have asthma if Beta-blockers are contraindicated, poorly tolerated, or do not control anginal symptoms. Angiotensin II receptor blockers can be given to the patient safely because they have no harmful effects on the respiratory system. p. 714

A patient reports nonradiating pain in the buttocks and numbness and tingling in the toes. The patient tells the nurse that the symptoms have been getting worse over the last few months, but the patient cannot remember what triggered it. The nurse suspects the patient may be experiencing which type of low back pain? A. Sciatica B. Chronic back pain C. Acute low back pain D. Overactivity back pain

Answer: B. Chronic back pain Rationale: Chronic back pain lasts more than three months, and the exact cause can be difficult to pinpoint. Sciatica is a type of radicular pain but does not match the subjective description of the patient's experience. Acute low back pain lasts 4 weeks or less; this patient has had pain for the last few months. Overactivity back pain does not fit the patient's description because exertion is a triggering factor for pain.

The nurse recognizes that a patient who regularly snorts cocaine is at risk for which complication? A. Gastric ulcers B. Chronic sinusitis C. Respiratory depression D. Impaired immune function

Answer: B. Chronic sinusitis Rationale: Cocaine users are at high risk for chronic sinusitis. Opioids cause gastric ulcers. Sedative-hypnotics are linked to respiratory depression. Cannabis may cause impaired immune function. p. 143

Which surgical interventions are often used for palliative care? Select all that apply. A. Biopsy B. Colostomy C. Tonsillectomy D. Laminectomy E. Insertion of a feeding tube F. Prophylactic mastectomy

Answer: B. Colostomy D. Laminectomy E. Insertion of a feeding tube Rationale: A colostomy, an insertion of a feeding tube, and a laminectomy are surgical procedures that provide supportive care to maximize bodily function without curing cancer on their own. A tonsillectomy is not a supportive surgical procedure; it is done to cure tonsil problems. A prophylactic mastectomy is a preventative measure. A biopsy is the removal of a tissue sample for pathologic analysis; it is diagnostic and would not benefit a patient receiving palliative care. p. 243

Which statement about chronic sinusitis is true? A. Severe pain occurs. B. Diagnosis is difficult. C. Nasal drainage is purulent. D. Fever starts before the infection.

Answer: B. Diagnosis is difficult. Rationale: Chronic sinusitis is hard to diagnose because symptoms are often nonspecific. Severe pain and purulent nasal drainage do not occur. Patients are rarely febrile. p. 485

The medical-surgical charge nurse receives a patient in the post anesthetic care unit after successful completion of spinal fusion. The patient is still sedated and is assigned to a staff nurse. The unit charge nurse intervenes when the staff nurse prepares to give which prescription to the patient? A. Morphine, 2 mg, intravenously (IV) B. Diazepam, 2.5 mg, by mouth C. Morphine via patient-controlled analgesia pump D. Hydromorphone, 1 mg, intravenously

Answer: B. Diazepam, 2.5 mg, by mouth Rationale: The sedated patient should not be given an oral medication. The presence of a gag reflex and the ability to tolerate oral fluids must be assessed first.

The nurse is providing health education to a patient who is scheduled for disc replacement. Which information should the nurse include in the teaching plan about the procedure? A. Disc replacement will permanently resolve pain. B. Disc replacement will reduce, but not eliminate, pain. C. Pain from the procedure should be resolved within five days. D. Disc replacement will allow the patient to regain hyperextension ability.

Answer: B. Disc replacement will reduce, but not eliminate, pain. Rationale: Disc replacement will likely make the pain more manageable, but it is not a permanent or long-term solution.

The nurse is caring for a patient who has undergone cervical spine surgery. Which findings should prompt the nurse to implement immediate life-saving measures? A. Cough B. Dyspnea C. Nasal flaring D. Vesicular breath sounds E. Diminished breath sounds

Answer: B. Dyspnea C. Nasal flaring Rationale: Dyspnea and nasal flaring are signs of respiratory distress and can be life threatening to the patient.

Which statement about influenza is true? A. The onset is insidious B. Generalized myalgia occurs. C. Vomiting and diarrhea result. D. Nuchal rigidity starts before headache.

Answer: B. Generalized myalgia occurs. Rationale: Generalized myalgia or body aches are common flu symptoms. The onset of flu is abrupt and not insidious. Anorexia occurs, but not vomiting and diarrhea. Nuchal rigidity is impaired neck flexion resulting from muscle spasms of the neck and is related to meningeal irritation. pp. 483-484

When the nurse is caring for a patient with chronic obstructive pulmonary disease (COPD) exacerbation who requires oxygen administration, which assessment finding is of most concern? A. Bilateral rhonchi B. Increased lethargy C. PaCO2 50 mm Hg D. O2 saturation 92%

Answer: B. Increased lethargy Rationale: Increased lethargy can be caused by high levels of CO2 since higher-than-normal PaCO2 levels cause central nervous system depression. Although patients with COPD exacerbation may require high oxygen flow rates if they are hypoxemia, the nurse needs to be aware that respiratory drive is sometimes decreased when oxygen saturations are increased. Rhonchi are an expected finding in a patient with a COPD exacerbation. Since air trapping and CO2 retention occurs in COPD, a PaCO2 slightly higher than normal is an expected finding for this patient. Although the oxygen saturation is in the low-normal range, this is an expected finding for a patient with a COPD exacerbation, and the oxygen saturation indicates adequate blood and tissue oxygenation. p. 1594

While giving medications, the nurse notices that a patient with asthma is not using the prescribed inhaler correctly. Which action will the nurse take first? A. Referral to an asthma support group B. Informal demonstration/return demonstation C. In-depth assessment of patient learning needs D. Development of a formal asthma teaching plan

Answer: B. Informal demonstration/return demonstration Rationale: Teaching about correct inhaler use is an opportunity for informal teaching, since the nurse is already administering the medication to the patient. Referral to an asthma support group may be needed in the future, but there is no indication that referral is needed at this time. An in-depth learning needs assessment may be needed in the future, but the nurse has already done the assessment needed to determine that the patient needs informal teaching. A formal asthma teaching plan may be needed at some point, but is not necessary to provide informal teaching. pp. 45-46

Which rationale supports a patient with severe chronic pain taking both an opioid and a non steroidal anti-inflammatory drug (NSAID) to relieve pain? A. It provides better pain relief to the patient. B. It prevents the side effects of the opioid drug. C. It manages the side effects of the opioid drug. D. It prevents the side effects of the NSAID drug.

Answer: B. It prevents the side effects of the opioid drug.

When assessing a patient receiving mechanical ventilatory support, which clinical manifestations would the nurse associate with ventilator-associated pneumonia? Select all that apply. A. Hypothermia B. Odorous sputum C. Crackles on auscultation D. Reduced white blood cell count E. Pulmonary infiltrates on a chest x-ray

Answer: B. Odorous sputum C. Crackles on auscultation E. Pulmonary infiltrates on a chest x-ray Rationale: Odorous sputum, crackles on auscultation, and pulmonary infiltrates noted on a chest x-ray are all clinical manifestations that suggest that the patient has ventilator-associated pneumonia. Hyperthermia (fever), not hypothermia, and elevated white blood cell count are other manifestations of ventilator-associated pneumonia. p. 1559

A patient is being seen in the emergency department for an acute asthma attack. Initial forced expiratory volume in 1 second (FEV1) is 70%. After the initial dose of albuterol, FEV1 is 75%. Which provider order would the nurse carry out first? A. Obtain STAT measurement of arterial blood gases (ABGs). B. Repeat albuterol dose. C. Obtain electrocardiogram. D. Administer pain medication.

Answer: B. Repeat albuterol dose. Rationale: The nurse would administer another dose of albuterol to continue to help reduce the patient's symptoms. Initially, three doses of albuterol are given in the ED depending on the patient's response to the medication.

Which type of medication may be included in the plan of care for a patient with chronic obstructive pulmonary disease (COPD) who experiences dyspnea and has a forced expiratory volume (FEV1) of 70% of predicted value? A. Inhalational budesonide B. Short-acting bronchodilator C. Long-acting theophylline (Theo-24) D. A combination of fluticasone and salmeterol

Answer: B. Short-acting bronchodilator Rationale: Since the patient has an FEV1 of 70% and is suffering from mild COPD, the patient may benefit from the use of short-acting bronchodilators. Budesonide, being a steroid, is not used for treating mild COPD. Use of theophylline in COPD is controversial and should be used only in patients who do not respond to other drugs. A combination of fluticasone and salmeterol is not required for treating mild COPD; this medication is prescribed to patients who have an FEV1 of less than 60%. p. 566

A patient is admitted to a general medical-surgical hospital unit from the home setting for the treatment of pneumonia. The patient receives a prescription for cefuroxime. The nurse verifies that which diagnostic study is completed before administering the first dose of the medication? A. Pulmonary function evaluation B. Sputum culture and sensitivity C. Chest x-ray D. Mantoux skin test

Answer: B. Sputum culture and sensitivity Rationale: The nurse should ensure that the sputum for culture and sensitivity was sent to the laboratory before administering the cefuroxime because this is community-acquired pneumonia. It is important that the organisms are identified correctly (by the culture) before the antibiotic takes effect. The test also will determine whether the proper antibiotic has been prescribed (sensitivity testing). Although antibiotic administration should not be unduly delayed while waiting for the patient to expectorate sputum, the pulmonary function evaluation and the chest x-ray will not be affected by the administration of antibiotics. The Mantoux test is used to determine the presence or absence of tuberculosis; it is read by inspection and palpation 48 to 72 hours later. With the information given, it would not be prescribed for this patient. p. 506

Which form of exercise will the nurse suggest for a patient with low back pain from a herniated lumbar disc? A. Yoga B. Walking C. Calisthenics D. Weightlifting

Answer: B. Walking

The nurse expects which medication may be prescribed for a patient who presents with fever, muscle aches, cough, sore throat, purulent sputum, and a positive rapid flu test? A. Zileuton B. Zanamivir C. Loratadine D. Clemastine

Answer: B. Zanamivir Rationale: A rapid flu test is performed in a patient suspected to have influenza, and the test suggests the presence of either the influenza A virus or the influenza B virus. Zanamivir is a neuraminidase inhibitor that acts by preventing the budding and spreading of the virus to adjacent cells. Zileuton is a leukotriene receptor inhibitor used to treat rhinitis or sinusitis. Loratadine is a second-generation antihistamine used to treat either rhinitis or sinusitis. Clemastine is a first-generation antihistamine used in the treatment of sinusitis. p. 485

The nurse expects which medication may be prescribed for a patient who presents with chills, a runny nose, a sore throat, purulent sputum, a cough, and crackles auscultated in the left lung base? A. Cetirizine B. Zanamivir C. Fluticasone D. Ciclesonide

Answer: B. Zanamivir Rationale: The patient with influenza experiences chills, rhinorrhea, sore throat, purulent sputum, cough, and pneumonia. Zanamivir is a neuraminidase inhibitor used to treat influenza and is likely to be prescribed by the health care provider. This medication acts by preventing the budding and spreading of the influenza virus to other cells. Cetirizine is a second-generation antihistamine used to treat rhinitis and sinusitis. Fluticasone and ciclesonide are corticosteroids used to treat rhinitis and sinusitis. p. 485

The nurse will include which teaching about dietary modifications for a patient with tonsillitis and sinusitis? A. "Drink herbal tea that contains kava." B. "Drink a glass of aloe vera juice daily." C. "Increase the intake of foods rich in zinc." D. "Add a small amount of garlic to your food."

Answer: C. "Increase the intake of foods rich in zinc." Rationale: Zinc is useful in the treatment of upper respiratory tract infections such as tonsillitis and sinusitis. Therefore the nurse advises the patient to increase the intake of food rich in zinc. Kava has anxiolytic action. It alleviates anxiety, not tonsillitis and sinusitis. Aloe increases peristalsis of the gastrointestinal tract and reduces the symptoms of constipation. Garlic lowers cholesterol and low-density-lipoprotein levels in the blood and reduces the risk of blood pressure; it will not help alleviate symptoms related to tonsillitis and sinusitis. p. 483

The nurse is providing orientation to a new graduate who is caring for a patient with a herniated disc. The graduate asks, "What is a disc herniation?" Which is the appropriate response by the nurse? A. "The annulus fibrous is deteriorating." B. "The annulus fibrous has penetrated the nucleus pulpous." C. "The nucleus pulpous has bulged out through an opening in the annulus fibrous." D. The nucleus pulpous is inflamed and causing irritation to the surrounding nerves."

Answer: C. "The nucleus pulpous has bulged out through an opening in the annulus fibrosus." Rationale: When the jellylike center of the vertebra bulges out through the tough outer layer of the vertebra, the disc is herniated."

A patient presents with signs and symptoms on pneumonia. What patient history should the nurse obtain to learn more about the type of pneumonia the patient may have? A. "Are you short of breath when sleeping?" B. "Have you ever had pneumonia in the past?" C. "When was the last time you were hospitalized?" D. "Has anyone in your family had pneumonia in the past?"

Answer: C. "When was the last time you were hospitalized?" Rationale: A patient with hospital-acquired pneumonia would be expected to be hospitalized within the last 48 hours, This question would provide the nurse important information to differentiate hospital-acquired pneumonia from community-acquired pneumonia.

The nurse is caring for a number of patients in the unit. Which patient most likely has chronic obstructive pulmonary disease (COPD)? A. A 33-year-old patient with a productive cough and yellow sputum. B. An 82-year-old patient with fever, fatigue, and low oxygen saturation. C. A 65-year-old smoker with 1:2 anterior-posterior (AP) chest diameter. D. A 45-year-old patient with dyspnea on exertion and bluish skin discoloration.

Answer: C. A 65-year-old smoker with 1:2 anterior-posterior (AP) chest diameter. Rationale: Smokers are at increased risk for COPD, and hyperinflation from COPD increases the patient's AP chest diameter.

A patient with osteoarthritis has been taking ibuprofen 400 mg every eight hours. The patient states that the drug does not seem to work as well as it used to in controlling the pain. Which information about non steroidal anti-inflammatory drugs (NSAIDs) would the nurse use as a basis of a response to the patient? A. It may take several months for NSAIDs to reach therapeutic levels in the blood. B. Patients often do not adhere to drug therapy with NSAIDs and other medications. C. A different NSAID may be indicated because of individual variations in responses. D. When NSAIDs are not effective, systemic corticosteroids are the next line of therapy.

Answer: C. A different NSAID may be indicated because of individual variations in responses. Rationale: Patients vary in their response to medications, so when one NSAID does not provide relief, another should be tried. There is no evidence to ascertain any non adherence with drug therapy. NSAIDs do not require extended time to reach therapeutic levels. Systemic corticosteroids are not the next in line for therapy; a different NSAID should be tried first. p. 112

A patient arrives at the emergency department stating that he twisted his back while playing football and is having "really bad pain." Which order does the nurse anticipate will be given initially for the patient? A. Apply ice pack for pain B. Restrict activity for 24 hours C. Administer ibuprofen for pain D. Refer patient for physical therapy

Answer: C. Administer ibuprofen for pain Rationale: Administration of a nonsteroidal anti-inflammatory drug, such as ibuprofen, is an appropriate initial pain relief measure for acute pain.

Which medication may prevent future attacks of gout for a patient who developed gout while hospitalized for a heart attack and who is currently taking aspirin? A. Amoxicillin B. Probenecid C. Allopurinol D. Nonsteroidal anti-inflammatory drugs (NSAIDs)

Answer: C. Allopurinol Rationale: To prevent future attacks of gout, the rate-lowering drug allopurinol may be administered. Treatment with amoxicillin has not been shown to have an effect on gout. This patient will not be able to take the uricosuric drug probenecid because the aspirin that the patient must take will inactivate its effect, resulting in rate retention. NSAIDs for pain management will not be used, related to the aspirin, because of the potential for increased side effects. p. 1515

Which respiratory defense mechanism may have failed in a patient who smokes one pack of cigarettes per day and has developed a diagnosis of pneumonia? A. Cough reflex B. Filtration of air C. Alveolar macrophages D. Mucociliary clearance system

Answer: C. Alveolar macrophages Rationale: Alveolar macrophages rapidly phagocytize inhaled foreign particles, such as bacteria, and often fail as a result of cigarette smoking. The cough reflex is not specifically affected by smoking. Filtration of air occurs in the nose, nasopharynx, and larynx and is not linked to a diagnosis of pneumonia. Patients with chronic obstructive pulmonary disease (COPD) and cystic fibrosis often have repeated lower respiratory tract infections as a result of mucocilary clearance failure. p. 459

The nurse is developing a teaching plan for a patient with newly diagnosed chronic obstructive pulmonary disease (COPD) who lives alone. The patient has a 30-pack-year history of cigarette smoking, walking 30 minutes twice per week, and is on a fixed income. Which part of the teaching plan is most important for the nurse to emphasize to the patient? A. Use of a topical nicotine patch B. Referral to a social worker for resources C. Assessment of readiness for smoking cessation D. Determination of the amount of supplemental oxygen to use during exercise

Answer: C. Assessment of readiness for smoking cessation Rationale: The nurse should assess the patient's readiness to begin smoking cessation before providing education or medication.

The nurse understands that which condition will cause worsening of symptoms of acute viral sinusitis? A. Influenza B. Fungal infection C. Bacterial infection D. Protozoal infection

Answer: C. Bacterial infection Rationale: Acute viral sinusitis may lead to secondary bacterial infection, which is manifested as high fever (more than 100.4 degrees Farenheit), swelling of the tonsils, severe ear pain, severe sinus pain, and worsening of present symptoms. Influenza, fungal, and protozoal infections are not caused by viral sinusitis. p. 485

Which parameter is most important to monitor for the patient who has been started on prednisone for an asthma exacerbation? A. Appetite B. Daily weight C. Breath sounds D. Blood glucose

Answer: C. Breath sounds Rationale: Because the patient is receiving an oral corticosteroid because of asthma exacerbation, the highest priority is to monitor for improvement in respiratory function. Corticosteroids may increase appetite and oral intake will be monitored, but the higher priority is assessment of respiratory status. Corticosteroids such as prednisone may lead to weight gain, but monitoring of weight is not as important as assessment of respiratory status in a patient with asthma. Blood glucose levels may increase while the patient is taking corticosteroids, but it is more important to evaluate for improvement in respiratory status. pp. 556-557

Nursing assessment findings of jugular venous distention and pedal edema would be indicative of what complication of chronic obstructive pulmonary disease (COPD)? A. Pneumonia B. Polycythemia C. Cor pulmonale D. Acute respiratory failure

Answer: C. Cor pulmonale Rationale: Cor pulmonale is a right-sided heart failure caused by resistance to right-ventricular outflow resulting from lung disease; clinical manifestations include jugular venous distention and pedal edema. Respiratory infections, such as pneumonia, can occur with COPD, but clinical manifestations would be fever and hemoptysis. Polycythemia is a complication of COPD, with clinical manifestations of increased hematocrit and cyanosis. Acute respiratory failure may occur with COPD, with symptoms of increasing dyspnea, hypoxemia, and hypercapnia. p. 563

The nurse is caring for a patient with chest tightness and difficulty breathing. The nurse notes accessory muscle use, wheezes, and a peak flow of 50%. After oral corticosteroids and a short-acting beta2-agonist have been administered, which assessment findings would indicate the need for further treatment? A. Temperature of 98.9 F B. Respiratory rate of 24 breaths/min C. Diminished breath sounds D. Moist mucous membranes

Answer: C. Diminished breath sounds Rationale: Diminished breath sounds indicate impeded air flow in the lungs and would require further treatment.

Which nursing assessment finding during an acute asthma attack suggests a serious complication requiring immediate intervention? A. Inspiratory wheezing B. Intercostal retractions C. Diminished breath sounds D. Hyperresonance percussion

Answer: C. Diminished breath sounds Rationale: Diminished breath sounds indicate poor air movement and require immediate intervention.

A patient with chronic obstructive pulmonary disease (COPD) presents with orthopnea and dyspnea that interferes with activities of daily living. On assessment, the forced expiratory volume in one second (FEV1) is 58%. Which medication would the nurse expect to administer as part of the maintenance treatment regimen? A. Albuterol B. Penicillin C. Fluticasone/salmeterol D. Budesonide/formoterol

Answer: C. Fluticasone/salmeterol Rationale: Fluticasone/salmeterol (sold commonly as Advair Diskus or Advair HFA) is combination therapy including an inhaled corticosteroid and a long-acting beta2 agonist (LABA) and is commonly used as maintenance therapy for patients with advanced COPD.

When a patient is experiencing an acute asthma attack, how will the nurse position the patient? A. Supine B. Lithotomy C. High-Fowler's D. Reverse Trendelenburg

Answer: C. High-Fowler's Rationale: The goal for positioning during an asthma attack is to maximize the patient's ability to inhale and exhale deep breaths, and the semi-to-high-Fowler's position will allow optimal chest expansion. When the patient is supine, the abdomen can push up against the chest and it is more difficult to fully expand the chest. The abdomen will place pressure against the chest in lithotomy position, making it more difficult to fully expand the lungs. While a reverse Trendelenburg position will decrease abdominal pressure on the chest, a more upright position is better for chest expansion. p. 557

Which interprofessional treatment will the nurse anticipate being prescribed for a patient who is seen in the emergency department for an asthma attack and has not improved after administration of albuterol? A. Fluticasone inhaler B. Salmeterol inhaler C. IV methylprednisolone D. Normal saline IV bolus

Answer: C. IV methylprednisolone Rationale: Systemic corticosteroids like IV methylprednisolone speed the resolution of asthma exacerbations and are indicated if the initial response to the short-acting Beta2-adrenergic agonist bronchodilator is insufficient. Inhaled corticosteroids such as fluticasone may be prescribed once the acute attack has resolved, but they take days or weeks to reach the maximum therapeutic level and would not be prescribed for an acute attack. Long-acting beta2-adrenergic agonists such as salmeterol may be added to the patient's medical regiment after the acute attack has resolved, but they would not be ordered to treat an acute attack. IV fluids may be prescribed, but they will not improve the inflammation and bronchoconstriction that are the cause of the asthma attack. p. 549

When a patient presents with acute exacerbation of asthma, which actions will the nurse anticipate as part of the inter professional treatment plan? Select all that apply. A. Administer antibiotics B. Give oral sedative medication. C. Infuse IV corticosteroids. D. Provide oxygen through a non-rebreather mask. E. Start nebulized short-acting beta2-adrenergic agonist.

Answer: C. Infuse IV corticosteroids. D. Provide oxygen through a non-rebreather mask. E. Start nebulizer short-acting beta2-adrenergic agonist. Rationale: Acute exacerbation of asthma may be life-threatening and needs immediate intervention. IV corticosteroids will act quickly to decrease airway inflammation. High oxygen concentrations are needed to avoid hypoxemia and can be administered using a non-rebreather mask. Inhaled short-acting beta2-adrenergic agonists will act quickly to dilate airways. Antibiotics are not administered unless there are symptoms of a respiratory infection. Sedatives are avoided, as they may depress the respiratory center and worsen hypoxemia. pp. 547-549

A patient with asthma has been taking low-dose inhaled corticosteroids along with long-acting Beta2-agonists for three months and shows signs of inadequate control. Which treatment would the nurse expect to incorporate into the plan of care? A. Leukotriene receptor antagonist B. Short-acting Beta2-agonist as needed C. Medium-dose inhaled corticosteroids and long-acting Beta2-agonist D. High-dose inhaled corticosteroids, long-acting Beta2-agonist, and oral corticosteroids

Answer: C. Medium-dose inhaled corticosteroids and long-acting Beta2-agonist Rationale: A patient who is treated with low-dose inhaled corticosteroids along with long-acting Beta2-agonists for asthma is receiving step 3 care to control exacerbations. If the asthma control is inadequate, the patient needs step 4 care to control the symptoms, which includes treatment with medium-dose inhaled corticosteroids along with a long-acting Beta2-agonist. Short-acting Beta2-agonists as needed are the treatment option for patients with intermittent asthma. Treatment with leukotriene receptor antagonists is a step 2 care option and is not effective for the patient who did not respond to step 3 care. Treatment with high-dose inhaled corticosteroids along with a long-acting Beta2-agonist and oral corticosteroids is step 6 care, which

The nurse is assessing the dressing of a patient after completion of lumbar spine surgery and notes yellow-tinged drainage. After further examination, the nurse documents that the patient has a headache with a rating of seven on a pain scale of one to 10. Which action is MOST appropriate action for the nurse to take next? A. Administer morphine intravenously B. Test the drainage for the presence of glucose C. Notify the health care provider immediately D. Remove the dressing and apply a pressure bandage

Answer: C. Notify the health care provider immediately Rationale: The health care provider should be altered immediately because these are signs of cerebrospinal fluid leakage, which can be life threatening.

The nurse anticipates which diagnosis for a patient exposed to animal dander who presents with a runny and itchy nose, sneezing, and nasal congestion? A. Sinusitis B. Influenza C. Perennial rhinitis D. Acute viral rhinitis

Answer: C. Perennial rhinitis Rationale: Rhinitis is the reaction of nasal mucosa when exposed to specific allergens. Perennial rhinitis is suspected when the patient has been exposed to animal dander, dust mites, indoor molds, or cockroaches. A patient with perennial rhinitis experiences sneezing, runny nose, nasal congestion, and watery eyers. A patient with sinusitis has symptoms of fever, malaise, purulent nasal drainage, nasal obstruction, and congestion. Influenza is a respiratory illness caused by the influenza virus. A patient with influenza presents with chills, fever, anorexia, malaise, sore throat, and pneumonia. Acute viral rhinitis is suspected when the patient has been in contact with another individual who has acute viral rhinitis. p. 480

During a routine wellness visit, a patient reports asthma symptoms occurring approximately three times per week. The symptoms do not interfere with the patient's sleep, and on assessment, forced expiratory volume in 1 second is within normal range. Which intervention can the nurse anticipate for this patient? A. Intravenous (IV) corticosteroids B. Antihistamine tablets C. Short-acting beta2 agonist D. Home oxygen supplementation

Answer: C. Short-acting beta2 agonist Rationale: A short-acting beta2-agonist would be given to patient with a mild asthma exacerbation.

When preparing to care for a patient with intervertebral disc disease, what should the nurse plan to tell the patient to do in order to maintain proper body mechanics? A. Use a backless chair B. Stand with feet close together C. Sit upright while seated at a desk D. Use a soft, airy mattress for sleep

Answer: C. Sit upright while seated at a desk Rationale: Sitting upright will promote and maintain proper alignment of the spine.

A patient with dyspnea and who is unable to speak clearly has a respiratory rate of 45 breaths/min, a pulse of 130 beats/min, oxygen saturation of 90%, and neck vein distention. Which treatment would the nurse expect to incorporate into the plan of care? A. Administration of ipratropium orally B. Three puffs of albuterol every 30 minutes C. Supplementary oxygen through nasal cannula D. Obtaining peak flow rate and monitoring the patient continuously

Answer: C. Supplementary oxygen through nasal cannula Rationale: The patient with a severe attack of asthma has an elevated respiratory rate, decreased oxygen saturation and elevated pulse, and the inability to speak, which indicate severe airway obstruction. The patient may also have neck vein distention. Hence, the nurse should anticipate the correction of hypoxemia and improvement of ventilation in the patient with supplementary oxygenation of nasal prongs. This helps to keep the oxygen saturation above 90%. Administration of ipratropium does not provide additional benefits to the patient. Three puffs of albuterol every 30 minutes will help to resolve the symptoms in patients with mild asthma. Obtaining the peak flow rate and continuous monitoring of the patient is critical during an asthma attack but will not alleviate the patient's symptoms. pp. 545-546

Which exercise will the nurse suggest to a patient with asthma, low back pain from a herniated lumbar disc, and hypertension? A. Tennis B. Running C. Walking D. Weightlifting

Answer: C. Walking

The nurse determines effective discharge teaching for a patient with pneumonia when the patient makes which statement? A. "I should seek immediate medical treatment for any upper respiratory infections." B. "I should continue to do deep-breathing exercises for at least 12 weeks." C. "I will increase my food intake to 2400 calories a day to keep my immune system well." D. "I will need to have a follow-up chest x-ray in six to eight weeks to evaluate the pneumonia's resolution."

Answer: D. "I will need to have a follow-up chest x-ray in six to eight weeks to evaluate the pneumonia's resolution." Rationale: The follow-up chest x-ray will be done in six to eight weeks to evaluate pneumonia resolution. A patient should seek medical treatment for upper respiratory infections that persist fro more than seven days. It may be important for the patient to continue with deep-breathing exercises for six to eight weeks, not 12 weeks, until all of the infection has cleared from the lungs. Increased fluid intake, not caloric intake, is recommended to liquefy secretions. p. 509

Which patient statement about the use of an ipratropium inhaler for chronic obstructive pulmonary disease (COPD) indicates a need for further teaching? A. "I can rinse my mouth following the two puffs to get rid of the bad taste." B. "I should wait at least one to two minutes between each puff of the inhaler." C. "Because this medication is not fast acting, I cannot use it in an emergency if my breathing gets worse." D. "If my breathing gets worse, I should keep taking extra puffs of the inhaler until I can breathe more easily."

Answer: D. "If my breathing gets worse, I should keep taking extra puffs of the inhaler until I can breathe more easily." Rationale: The patient should not just keep taking extra puffs of the inhaler to make breathing easier. Excessive treatment could trigger paradoxical bronchospasm, which would worsen the patient's respiratory status. Rinsing the mouth after the puffs will eliminate a bad taste. Waiting one to two minutes between each puff will facilitate the effectiveness of the administration. Ipratropium is not used in an emergency for COPD. pp. 551-554

A patient being prepared for a laminectomy tells the nurse, "I only took six days off from work to recover. Will that be enough time?" Which response by the nurse is appropriate? A. "You should have at least 10 days off from work to recover." B. "You should speak to the surgeon about how much recovery time is needed." C. "As long as you will only be sitting down and not moving around much, you should be fine." D. "It may take a few weeks to fully recover from the surgery. Six days may not be enough time."

Answer: D. "It may take a few weeks to fully recover from the surgery. Six days may not be enough time." Rationale: Depending on the amount of lifting, walking, and sitting the patient's job involves, the patient may be able to return to work within a few weeks.

During a triage assessment, the nurse notices that a patient with low back pain is sobbing softly. Based on this observation, which would be an appropriate response? A. "You seem upset. I'll leave you alone for a while." B. "Don't be upset. Many people suffer with back pain." C. "Everything will be all right. We will find a treatment that works." D. "You are crying. Do you want to talk about how you're feeling?"

Answer: D. "You are crying. Do you want to talk about how you're feeling?"

The nurse is caring for a patient with unrelieved chronic pain. The patient becomes very tearful and says, "No amount of physical therapy or drugs has worked. I'll never get relief!" Which is an appropriate response by the nurse? A. "Don't worry. It can take some time for your pain to be relieved." B. "I know exactly how you feel, but there are other things that can be done." C. "I can speak to your health care provider about increasing your pain medication dose." D. "You seem very upset. Would you like to talk to your provider about surgical options?"

Answer: D. "You seem very upset. Would you like to talk to your provider about surgical options?" Rationale: This response by the nurse acknowledges the patient's feelings and offers the option of exploring other possible methods of treatment. The nurse understands that several surgical options may be used to treat chronic back pain that does not respond to noninvasive therapy.

Which patient does the nurse identify as having the greatest risk for developing low back pain? A. A 24-year-old woman five months postpartum B. A 41-year-old patient with a history of a tibia fracture C. A 56-year-old patient who smoked during adolescence D. A 30-year-old patient with a body mass index (BMI) 10% above recommended value

Answer: D. A 30-year-old patient with a body mass index (BMI) 10% above recommended value. Rationale: Being overweight is a risk factor for low back pain, and this patient has a BMI 10% above the recommended value.

The nurse assesses a patient with wheezing and dyspnea and determines that these symptoms are related to which pathophysiologic feature asthma? A. Laryngeal stridor B. Alveolar collapse C. Mucus production D. Bronchoconstriction

Answer: D. Bronchoconstriction Rationale: Narrowing (constriction) of the airway leads to reduced airflow, making it difficult for the patient to breathe and producing the characteristic wheezing. Mucus production may occur but is not responsible for wheezing. Alveolar collapse and laryngeal stridor are not related to asthma. p. 543

When caring for a patient with chronic obstructive pulmonary disease (COPD), which assessment finding will the nurse expect? A. Sore throat B. Pleuritic chest pain C. Elevated temperature D. Chronic intermittent cough

Answer: D. Chronic intermittent cough Rationale: The earliest clinical manifestation of COPD is usually a chronic intermittent cough. Sore throat is not typical for COPD but would suggest an upper respiratory process, such as pharyngitis. Pleuritic chest pain is typical of infectious or inflammatory conditions, such as pneumonia or pleural effusion. Elevated temperature is not expected with COPD unless there is a complication, such as a viral or bacterial respiratory infection. p. 563

A patient presents with a pneumonia score of 5 on the Expanded CURB-65 scale. Which action does the nurse take? A. Advise no treatment. B. Advise treating in an outpatient setting. C. Consider admission to an inpatient medical-surgical unit. D. Consider admission to an intensive care unit.

Answer: D. Consider admission to an intensive care unit. Rationale: The Expanded CURB-65 scale may be used as a supplemental to clinical judgment to determine the severity of pneumonia and if patients need to be hospitalized. A patient score of 5 on the scale means the perceived risk is high and that placement in the intensive care unit is warranted. If the patient has symptoms of pneumonia, advising no treatment is not appropriate. Treatment in an outpatient setting is advised when the scores are 0 to 2 on the scale. Hospital admission is advised when the scores are 3 to 4 on the scale. p. 504

Which explanation is likely when the nurse notes a hemoglobin level of 20 g/dL in a patient admitted with chronic obstructive pulmonary disease (COPD)? A. COPD has resolved. B. Patient's diet is high in iron. C. Cardiac function has improved. D. Hypoxia has stimulated erythropoiesis.

Answer: D. Hypoxia has stimulated erythropoiesis. Rationale: In COPD, there is chronic hypoxia, which causes a compensatory increase in red blood cell production and leads to polycythemia. Since COPD is irreversible, the patient still has COPD, and the elevated hemoglobin level is consistent with the COPD diagnosis. Dietary intake of iron-rich foods is unlikely to lead to such a high hemoglobin level. Cardiac output may be adversely affected by the increase in blood viscosity associated with polycythemia. pp. 563-564

A patient presents with nasal flaring, intercostal retractions, and chest tightness. The nurse notes diminished breath sounds over both lungs. After administering albuterol and intravenous magnesium sulfate, which assessment finding indicates treatment has been effective? A. Heart rate of 122 beats/min B. Persistent barking cough C. Supraclavicular retratctions D. Inspiratory and expiratory wheezing

Answer: D. Inspiratory and Expiratory wheezing Rationale: Inspiratory and expiratory wheezing heard after treatment indicates improved movement of air in the lungs and effectiveness of treatment.

The nurse expects which anticholinergic nasal spray may be prescribed for a patient with nonallergic rhinitis? A. Phenylephrine B. Triamcinolone C. Cromolyn spray D. Ipratropium bromide

Answer: D. Ipratropium bromide Rationale: Ipratropium bromide blocks nasal cholinergic receptors, reducing nasal secretions in the common cold and nonallergic rhinitis. Phenylephrine is a decongestant nasal spray used to treat rhinitis and sinusitis. Triamcinolone is a corticosteroid nasal spray used to treat sinusitis. Cromolyn spray is a mast cell stabilizer nasal spray used to treat rhinitis and sinusitis. p. 481

Which surgical option may be utilized to reduce the size of hyperinflated emphysematous lungs and decrease airway obstruction for a patient with severe chronic obstructive pulmonary disease (COPD)? A. Bullectomy B. Airway bypass C. Lung transplantation D. Lung volume reduction surgery (LVRS)

Answer: D. Lung volume reduction surgery (LVRS) Rationale: Three different surgical procedures have been used in severe COPD. One type of surgery is LVRS. The goal of this surgery is to reduce the size of the lungs by removing the most diseased lung tissue so the remaining healthy lung tissue can perform better. The rationale for LVRS is that reducing the size of the hyper inflated emphysematous lungs results in decreased airway obstruction and increased room for the remaining normal alveoli to expand and function. The procedure reduces lung volume and improves lung and chest wall mechanics. Bullectomy is for patients with emphysematous COPD who have large bullae. Airway bypass is a procedure still under evaluation. Lung transplantation would not be the first choice, because it is for carefully selected patients with advanced COPD, and this patient's lung volume can be reduced. p. 566

In which position will the nurse place the patient with right-sided pneumonia to improve pulmonary gas exchange? A. Prone position B. Supine position C. Tripod position in bed D. Lying on left side position

Answer: D. Lying on left side position Rationale: Because pulmonary ventilation and perfusion are improved in dependent areas of the lungs, oxygenation will be maximized when a patient with a ride-sided pneumonia is positioned with the good lung down (I.e., lying on left side position). The prone position is sometimes helpful for patients with acute respiratory distress syndrome but is not recommended for patients with a one-sided respiratory infectious process. The supine position makes it difficult for patients to take deep breaths and predisposes to atelectasis. The tripod position helps to increase chest and lung expansion in patients with chronic obstruction pulmonary disease but would not be especially helpful for patients with right-sided pneumonia. p. 1595

Which symptoms indicate that a patient with an upper respiratory infection (URI) may have developed acute sinusitis? A. Coughing B. Fever, chills C. Dust allergy D. Maxillary pain

Answer: D. Maxillary pain Rationale: The nurse should assess the patient for sinus pain or pressure as a clinical indicator of acute sinusitis. Coughing and fever are nonspecific clinical indicators of a URI. A history of an allergy that is likely to affect the upper respiratory tract is supportive of the sinusitis diagnosis, but is not specific for sinusitis. p. 485

Which finding is the best indicator that a patient with viral pneumonia is developing acute respiratory distress syndrome (ARDS)? A. Crackling sounds in lung bases B. Cough productive of thick green mucus C. Intercostal retractions noted bilaterally with inspiration D. No improvement in O2 saturation with oxygen administration

Answer: D. No improvement in O2 saturation with oxygen administration Rationale: Refractory hypoxemia is the hallmark characteristic of ARDS. Crackles in the lung bases may occur in the early stages of ARDS, but crackles could also be caused by atelectasis or be a symptom of the patient's viral pneumonia. A productive cough is more typical of pneumonia than of ARDS. Intercostal retractions occur with increased work of breathing in ARDS but may also be present with viral pneumonia. p. 1599

The nurse learns that a patient with asthma has a three-year history of chronic sinusitis. Which action by the nurse is appropriate? A. Ask the health care provider to add a Beta-antagonist medication. B. Encourage adherence to an avoidance diet to decrease asthma triggers. C. Advise the patient to exercise when the air is cool and dry to prevent exacerbations. D. Obtain a surgeon referral.

Answer: D. Obtain a surgeon referral. Rationale: Both acute and chronic sinusitis make asthma worse, causing severe inflammation of the mucous membranes. Such patients should undergo the removal of large nasal polyps to help control exacerbations of asthma. Beta-Antagonists promote bronchoconstriction, leading to further airway obstruction and breathlessness in the patient. The nurse should advise the patient to avoid exercise when the air is cool and dry to prevent exacerbations. Avoidance diets are not recommended until testing has proven an allergy is present. pp. 542-543

A patient with human immunodeficiency virus (HIV) infection has been admitted with penumonia. The nurse anticipates that the patient will receive treatment for which type of pneumonia? A. Aspiration pneumonia B. Hospital-associated pneumonia C. Community-acquired pneumonia D. Opportunistic pneumonia caused by Pneumocystis jiroveci

Answer: D. Opportunistic pneumonia caused by Pneumocystis jiroveci Rationale: Individuals at risk for opportunistic pneumonia include those with altered immune responses, such as HIV infection. In addition to the risk of bacterial and viral pneumonia, the immunocompromised person may develop an infection from microorganisms that do not normally cause disease, such as P. jiroveci (formerly carinii). The patient likely does not have aspiration pneumonia, hospital-associated pneumonia, or community-acquired pneumonia. pp. 504-505

The nurse expects which medication may be prescribed for a patient who reports pain at the maxillary sinuses, headache, nasal obstruction, discolored purulent nasal drainage, and whose CT scan shows the presence of thickened mucous membrane at the maxillary sinuses? A. Nystatin B. Zanamivir C. Oseltamivir D. Pseudoephedrine

Answer: D. Pseudoephedrine Rationale: Sinusitis is an inflammation of the mucosa that blocks the opening of the Ostia in the sinuses through which the mucus drains into the nose. Pseudoephedrine is a decongestant used in the treatment of sinusitis. It works by stimulating the adrenergic receptors on the blood vessels and reduces nasal congestion by promoting vasoconstriction. Nystatin is an anti fungal medication used to treat acute pharyngitis caused by Candida infections. Zanamivir and oseltamivir are neuraminidase inhibitors used to treat influenza. p. 481

A patient with asthma presents with complaints of dyspnea on exertion. The nurse notes hyper resonance on percussion. Which action should the nurse take? A. Administer albuterol inhaler. B. Administer fluticasone tablet. C. Obtain a STAT chest x-ray. D. Teach the patient the pursed-lip breathing technique.

Answer: D. Teach the patient the pursed-lip breathing technique. Rationale: Hyperresonance on percussion indicates air trapping. The nurse should teach the patient the pursed-lip breathing technique to prevent hyperinflation.

A patient with asthma develops nausea, vomiting, headache, tachycardia, and dysrhythmias. The nurse would suspect which medication as the cause of the symptoms? A. Albuterol B. Omalizumab C. Mometasone D. Theophylline

Answer: D. Theophylline Rationale: The patient with asthma may develop nausea, vomiting, headache, tachycardia, and dysrhythmias upon theophylline use. Theophylline is a methylxanthine bronchodilator with narrow therapeutic use. Therefore its use is very limited. The patient's serum concentration levels should be monitored regularly to determine that the drug is within the therapeutic window. Albuterol is a short-acting beta-agonist that is safe in the patient with mild asthma. Mometasone is a corticosteroid that may cause local irritation, such as cough and hoarseness upon inhalation. Omalizumab is a monoclonal antibody that decreases circulating free immunoglobulin E (IgE) levels in the patient with allergic asthma. p. 550

Which exercise goal is most appropriate for a patient with chronic obstructive pulmonary disease (COPD)? A. Slight increase in activity over the current level B. Limitation of exercise to activities of daily living to conserve energy C. Swimming for 10 minutes/day, gradually increasing to 30 minutes/day D. Walking for 20 minutes/day, keeping the pulse rate less than 130 beats/minute

Answer: D. Walking for 20 minutes/day, keeping the pulse rate less than 130 beats/minute Rationale: The patient will benefit from mild aerobic exercise that does not stress the cardiorespiratory system. The patient should be encouraged to walk for 20 minutes/day and should be encouraged to decrease activity if it takes longer than five minutes to return to baseline. Activity should be steadily increased according to patient tolerance. Regular physical exercise is important for patients with COPD and should include more than just activities of daily living. Swimming may be too taxing for the patient with COPD; these patients should exercise using stationary cycling or walking. p. 575

A patient presents to the emergency department with dyspnea. The patient's vital signs are temperature 98.6 F, blood pressure 160/100 mm Hg, heart rate 90 beats/minute, respiratory rate 25 breaths/min, and functional oxygen saturation of 80%. Peak expiratory flow rate (PERF) is 52%. Which orders should the nurse prioritize for this patient? Select all that apply. A. Albuterol B. Oral aspirin C. Oral metoprolol D. Supplemental oxygen E. Intravenous magnesium sulfate

Answers: A. Albuterol D. Supplemental oxygen Rationale: Albuterol, a short-acting beta2-agonist, would be given to a patient with an acute asthma exacerbation. Supplemental oxygen should be administered to treat the patient's hypoxemia.

Which clinical manifestations indicate an exacerbation of chronic obstructive pulmonary disease (COPD)? Select all that apply. A. Arterial pH 7.26 B. PaCO2 42 mm Hg C. Patient in tripod position D. Increased difficulty sleeping E. Increased sputum expectoration

Answers: A. Arterial pH 7.26 C. Patient in tripod position D. Increased difficulty sleeping E. Increased sputum expectoration Rationale: An exacerbation of COPD is an acute event characterized by a worsening of the patient's respiratory symptoms. Exacerbations are signaled by an acute change in the patient's usual dyspnea, cough, and/or sputum. The patient's pH shows acidosis that supports an exacerbation of COPD along with the worsening dyspnea. A patient in a tripod position indicates he or she may be experiencing increased dyspnea. The nurse will assess patients for the classic manifestations of exacerbation, including increased dyspnea, increased sputum volume, or increased sputum purulence. They may have malaise, insomnia, fatigue, depression, confusion, decreased exercise tolerances increased wheezing, or fever. The PaCO2 is within normal limits. p. 564

Which interventions may be included in the pain of care for a patient with advanced stage chronic obstructive pulmonary disease (COPD) who has experienced significant weight loss? Select all that apply. A. Encourage rest before meals. B. Encourage high fluid intake with food. C. Discontinue oxygen therapy while eating. D. Use steroids and theophylline to relieve bloating. E. Encourage high-protein supplements between meals.

Answers: A. Encourage rest before meals. E. Encourage high-protein supplements between meals. Rationale: Decreased appetite and weight loss occur because of systemic inflammatory processes in COPD. Resting before taking meals helps the patient reduce dyspnea and conserve energy. High-protein supplements can be taken between meals to compensate for the catabolic effects of COPD. Fluid intake should be encouraged between meals rather than with food to prevent distention of the stomach. Supplemental oxygen therapy while eating is beneficial to the patient. Steroids and theophylline cause bloating and do not relieve it. p. 572

Which topics will be included when the nurse is teaching a patient with lung hyperinflation caused by severe chronic obstructive pulmonary disease (COPD)? Select all that apply. A. Huff coughing B. Low-calorie diet C. Smoking cessation D. Pursed-lip breathing E. Diaphragmatic breathing F. Use of airway clearance devices

Answers: A. Huff coughing C. Smoking cessation D. Pursed-lip breathing F. Use of airway clearance devices Rationale: Huff coughing is an effective coughing technique that helps to clear sputum from the airways. Smoking cessation prevents the accelerated decrease in pulmonary function that leads to COPD symptoms. Pursed-lip breathing improves expiratory function and decreases air trapping. Airway clearance devices help to clear respiratory secretions. Patients with COPD are encouraged to have a high-calorie diet to help meet metabolic needs caused by the increased work of breathing. Diaphragmatic breathing is designed to increase inspiratory volumes and may increase hyperinflation and the work of breathing in the patient who already has lung hyperinflation. pp. 566, 570-572

The nurse is caring for a patient who receives enteral feeding through a nasogastric (NG) tube. Which actions does the nurse perform to prevent aspiration in this patient? Select all that apply. A. Monitor gastric residual volumes. B. Assess the gag reflex before giving foods or fluids by mouth. C. Administer the initial feeding at a slow rate. D. Elevate the head of the bed to at least 30 degrees. E. Encourage the patient to sit upright for all meals.

Answers: A. Monitor gastric residual volumes. B. Assess the gag reflex before giving foods or fluids by mouth. D. Elevate the head of the bed to at least 30 degrees. E. Encourage the patient to sit upright for all meals. Rationale: Aspiration pneumonia results from the abnormal entry of material from the mouth or stomach into the trachea and lungs. Although feeding tubes are small, any interruption in the integrity of the lower esophageal sphincter can allow reflux of gastric contents. Conditions that increase the risk for aspiration include the insertion of NG tubes with or without enteral feeding. Gastric residual volumes should be monitored to determine how well the patient is tolerating the feeding. The nurse should assess for a gag reflex before giving good or fluids by mouth. For the patient who has difficulty swallowing and needs aid in eating, drinking, and taking medication to prevent aspiration, the nurse should elevate the patient's head of the bed to at least 30 degrees and have the patient sit up for all meals. The health care provider will prescribe the rate at which the enteral feeding is to be administered. p. 509

Which assessment findings correlate with a diagnosis of influenza? Select all that apply. A. Muscle aches B. Sore throat C. Temperature of 102.4 F D. Purulent nasal discharge E. Difficulty breathing through the nose

Answers: A. Muscle aches B. Sore throat C. Temperature of 102.4 F Rationale: Influenza is an infectious disease caused by a virus that attacks the respiratory system. Myalgia, sore throat, fever, chills, cough, and rhinorrhea are symptoms of influenza. Purulent nasal discharge and nasal obstruction are symptoms of sinusitis. p. 483

The school nurse will include which teaching to high school students to prevent the transmission of influenza? Select all that apply. A. Obtain an influenza vaccination. B. Stay at home when symptomatic. C. Drink non caffeinated fluids daily. D. Obtain antibiotic therapy promptly. E. Cover the nose and mouth when coughing.

Answers: A. Obtain an influenza vaccination. B. Stay at home when symptomatic. E. Cover the nose and mouth when coughing. Rationale: Covering the nose and mouth when coughing is an effective way to prevent the spread of the virus. Obtaining an influenza vaccination helps prevent the flu. Staying at home helps prevent direct exposure of others to the virus. Drinking fluids helps liquefy secretions but does not prevent influenza. Antibiotic therapy is not used unless the patient develops a secondary bacterial infection. pp. 483-484

Which nursing assessment findings would indicate that the patient has severe chronic obstructive pulmonary disease (COPD)? Select all that apply. A. Oxygen saturation of 82% B. Forced expiratory volume in 1 second (FEV1) of 39% C. Elevated liver enzyme levels D. Positive sputum culture E. Heart murmur on auscultation

Answers: A. Oxygen saturation of 82% B. Forced expiratory volume in 1 second (FEV1) of 39% Rationale: An oxygen saturation of 82% is hypoxemia, a sign of advanced COPD. An FEV1 between 30% and 49% indicates severe COPD. Elevated liver enzyme levels suggest liver failure, not advanced COPD. A positive sputum culture indicates infection, not advanced COPD. A heart murmur is not an indication of COPD.

Which medication instructions does the nurse provide for a patient with osteoarthritis who is initiating treatment with celecoxib? Select all that apply. A. Report persistent headaches B. Limit sodium intake C. Take with food or milk. D. Monitor BP. E. Have potassium levels checked regularly. F. Report tarry stools or bruising.

Answers: A. Report persistent headaches. C. Take with food or milk. D. Monitor BP. F. Report tarry stools or bruising.

The nurse cares for a 75-year-old patient with pneumonia and identifies that the patient is at risk for which complications? Select all that apply. A. Sepsis B. Pleurisy C. Bronchitis D. Encephalitis E. Pleural effusion F. Congestive heart failure

Answers: A. Sepsis B. Pleurisy E. Pleural effusion Rationale: Complications from pneumonia develop more often in older adults and those with underlying chronic diseases. These include sepsis/septic shock, which can occur when bacteria within the alveoli enter the bloodstream; pleurisy (an inflammation of the pleura); and pleural effusion (fluid in the pleural space). Bronchitis and encephalitis are not complications. Congestive heart failure is not directly known to be a complication of pneumonia. p. 506

The nurse will include which teaching for a patient with sinusitis? Select all that apply. A. Take plenty of rest. B. Drink plenty of water. C. Take a cool-water bath twice a day. D. Sleep with the head in a lower position. E. Perform large-volume nasal saline washes once or twice a day.

Answers: A. Take plenty of rest. B. Drink plenty of water. E. Perform large-volume nasal saline washes once or twice a day. Rationale: The nurse instructs a patient with sinusitis to take adequate rest because it prevents fatigue and helps the body fight against infection-causing bacteria. The nurse also recommends that the patient drink plenty of water and remain hydrated in order to loosen nasal secretions. The nurse recommends that the patient perform large-volume nasal saline washes once or twice a day in order to facilitate the removal of nasal fluids. The nurse should suggest that the patient take hot showers twice a day to provide comfort. The patient should sleep with his or her head elevated because it helps drain the sinuses. p. 486

Which non pharmaceutical interventions may help to manage pain for a patient with osteoarthritis? Select all that apply. A. Use of paraffin baths or hot packs B. Immobilization of the affected joint C. Use of crutches or walker if required D. Regulation of a normal body mass index (BMI) E. Strenuous exercise to keep the joints functional

Answers: A. Use of paraffin baths or hot packs C. Use of crutches or walker if required D. Regulation of a normal body mass index (BMI)

A patient with asthma is prescribed formoterol. Which directive would the nurse include in the medication education provided to the patient? Select all that apply. A. "Take the medication once every 24 hours." B. "Side effects include cold or flu-like symptoms." C. "Formoterol is the best medication to take to obtain quick relief from bronchospasm." D. "Formoterol is not the only medication that you will need to treat your asthma." E. "If wheezing gets worse, there are other types of medications that are more beneficial than formoterol."

Answers: B. "Side effects include cold or flu-like symptoms." D. "Formoterol is not the only medication that you will need to treat your asthma." E. "If your wheezing gets worse, there are other types of medications that are more beneficial than formoterol." Rationale: Formoterol is a long-acting Beta2-adrenergic agonist (LABA) and is effective in treating symptoms of asthma that persist at night and disrupt sleep. Formoterol is not helpful to treat acute exacerbations of worsening wheezing. In those cases, short-acting Beta2-adrenergic blockers may be more beneficial. Side effects of LABAS include cold or flu-like symptoms. LABAs should never be used as monotherapy for asthma, and they should only be used if the patient is also taking an inhaled corticosteroid. Short-acting Beta2-adrenergic blockers help the patient obtain quick relief from bronchospasm. The patient must take the drug every 12 hours. The nurse should instruct the patient not to overuse formoterol because there are other options also available. p. 550

The nurse is preparing to discharge a patient with chronic obstructive pulmonary disease (COPD) who reports weight loss of 10 lb in the past month, fatigue, and dyspnea at rest. Which recommendations should the nurse include in discharge teaching? A. Walking B. Adequate sleep C. Strength training D. High-calorie diet E. Oxygen supplementation

Answers: B. Adequate sleep D. High-calorie diet E. Oxygen supplementation Rationale: Adequate sleep is necessary to ensure the patient has energy and is able to perform activities of daily living. A high-calorie, high-protein diet and will promote energy. Oxygen supplementation treats hypoxia, which is a priority when the patient has dyspnea at rest.

A patient with lung cancer has intense, localized, persistent back pain and motor and sensory disturbances. Which interventions may be included in this patient's collaborative plan of care? Select all that apply. A. Withhold narcotics. B. Administer corticosteroids. C. Prepare the patient for a laminectomy. D. Prepare the patient for radiation therapy. E. Encourage a graded increase in patient activity.

Answers: B. Administer corticosteroids. C. Prepare the patient for a laminectomy. D. Prepare the patient fro radiation therapy.

A patient is experiencing an acute asthma attack. The nurse understands that which common triggers may be the cause of this attack? Select all that apply. A. Walking inside B. Allergic reaction to cats C. Intake of dairy products D. Exposure to air pollution E. Prolonged heat exposure

Answers: B. Allergic reaction to cats C. Intake of dairy products D. Exposure to air pollution Rationale: Allergic asthma can result from exposure to allergens. Dairy products can cause excessive mucus production and lead to asthma exacerbations in patients with asthma. Exposure to air pollution can lead to lung irritation and an acute asthma attack.

Which interventions help to reduce the incidence of pain for a patient with low back pain? Select all that apply. A. Maintaining the patient on strict bed rest B. Applying heat or ice to the area of most pain C. Preventing the patient from lifting anything at the bedside D. Administering prescribed non steroidal anti-inflammatory drugs (NSAIDs) E. Placing a pillow between the patient's legs and turning the body as a unit

Answers: B. Applying heat or ice to the area of most pain D. Administering prescribed non steroidal anti-inflammatory drugs (NSAIDs) E. Placing a pillow between the patient's legs and turning the body as a unit

A patient diagnosed with chronic obstructive pulmonary disease (COPD) experiencing shortness of breath and a cough asks the nurse the best way to prevent exacerbations. Which recommendations should the nurse make? A. Antibiotic therapy B. Effective handwashing C. Pursed-lipped breathing D. Oxygen supplementation E. Preventative vaccinations

Answers: B. Effective handwashing E. Preventative vaccinations Rationale: Effective handwashing can prevent secondary infections and prevent acute exacerbations. Preventative vaccinations can decrease the risk for secondary infections and prevent acute exacerbations.

The nurse understands that patients working in which occupations may have an increased developing chronic obstructive pulmonary disease (COPD)? A. Paramedics B. Exterminators C. Postal workers D. City bus drivers E. Construction workers

Answers: B. Exterminators D. City bus drivers E. Construction workers Rationale: Exterminators, city bus drivers, and construction workers are at increased risk for developing COPD because they are frequently exposed to dust, vapors, chemical irritants, and fumes.

A patient with chronic obstructive pulmonary disease (COPD) reports a chronic productive cough and an unintended weight loss of 15 lb over the previous month. Which recommendations should be made to this patient to address the current symptoms? A. Oral antibiotics B. High protein diet C. Oxygen supplementation D. Increased oral fluid intake E. Use of diaphragmatic breathing

Answers: B. High protein diet D. Increased oral fluid intake E. Use of diaphragmatic breathing Rationale: A diet that is high in calories and protein should be recommended because additional calories are needed to support increased energy expenditure and treat the patient's weight loss. Oral fluid intake thins lung secretions for better expectoration to treat the productive cough. Diaphragmatic breathing helps the patient breathe deeply (from the diaphragm) and can assist with clearing fluid and secretions from the airways.

Which aspects of the immune response are thought to be involved in the development of asthma? Select all that apply. A. Antibody formation B. Lack of conditioning C. Drainage of lymph fluid D. Immunoglobulin E (IgE) response to allergens E. Passive immune response

Answers: B. Lack of conditioning D. Immunoglobulin E (IgE) response to allergens

Which actions will the nurse include in the plan of care for a patient with respiratory failure caused by an exacerbation of chronic obstructive pulmonary disease (COPD)? Select all that apply. A. Elevate the food of the bed. B. Offer fluids at frequent intervals. C. Provide pharyngeal suctioning every 2 hours. D. Assist patient with use of albuterol nebulizer. E. Titrate oxygen to keep O2 saturation at 90%. F. Teach patient how to perform huff coughing.

Answers: B. Offer fluids at frequent intervals. D. Assist patient with use of albuterol nebulizer. E. Titrate oxygen to keep O2 saturation at 90%. F. Teach patient how to perform huff coughing. Rationale: Because a patient with COPD will have difficulty clearing thick secretions, offering fluids at frequent intervals and huge coughing are used to help to clear the airways. Short acting bronchodilators are used to decreased bronchospasm and improve ventilation. Oxygen administration is adjusted to keep the oxygen saturation in the low normal range, which will avoid respiratory depression. The head of the bed will be elevated in order to maximize respiratory expansion. Pharyngeal suctioning is used only if the patient is unable to clear respiratory secretions by coughing and is not scheduled every two hours. pp. 1594-1596

The nurse is caring for a patient with new-onset chronic obstructive pulmonary disease (COPD). Which assessment findings would be expected? Select all that apply. A. Dyspnea at rest B. Persistent cough C. Increased red blood cell (RBC) count D. Decreased anterior-posterior diameter of chest E. Weight loss of 10 lb in 1 month

Answers: B. Persistent cough C. Increased red blood cell (RBC) count E. Weight loss of 10 lb in 1 month Rationale: Persistent cough is a symptom of chronic bronchitis. An increased number of RBCs is indicative of polycythemia, which occurs as a result of hypoxemia in patients with COPD. Unexplained weight loss occurs in patients with COPD as a result of an increase in energy expenditure.

The nurse is caring for a patient with a history of asthma who reports severe dyspnea and the presence of chest tightness for 3 days that has escalated in the last few hours. The nurse notes, functional oxygen saturation of 87%, inspiratory and expiratory wheezes, and forced expiratory volume in 1 second (FEV1) of 23%. Which nursing actions are appropriate for this patient? Select all that apply. A. Administer albuterol. B. Provide supplemental oxygen. C. Administer intravenous (IV) corticosteroids. D. Give oral pain medication. E. Administer intravenous (IV) anti anxiety medication.

Answers: B. Provide supplemental oxygen. C. Administer intravenous (IV) corticosteroids. Rationale: The patient's symptoms indicate a moderate, acute asthma exacerbation. Supplemental oxygen is administered to treat the patient's hypoxemia. The patient's symptoms indicate a moderate, acute asthma exacerbation

When educating a patient about nonpharmacologic interventions to manage sinusitis, which instructions will the nurse include? Select all that apply. A. Restrict fluid intake. B. Use a steam inhaler. C. Avoid exposure to smoke. D. Sleep with your head elevated. E. Apply a cold compress on your cheeks.

Answers: B. Use a steam inhaler. C. Avoid exposure to smoke. D. Sleep with your head elevated. Rationale: In the case of sinusitis, steam inhalation helps to promote drainage of secretions. Sleeping with the head elevated helps to drain the sinuses and reduce congestion. Smoke is an irritant and will worsen the symptoms of sinusitis. Adequate fluid intake will decrease the symptoms of sinusitis. Applying a cold compress on the cheeks is not recommended because this worsens the symptoms. A hot compress on the cheeks will help. p. 486

Pneumonia will likely present with which breath sounds? Select all that apply. A. Stridor B. Wheezes C. Egophony D. Bronchophony E. Whispering pectoriloquy

Answers: C. Egophony D. Bronchophony E. Whispering pectoriloquy Rationale: Pneumonia will present with egophony, bronchophony, and whispering pectoriloquy. Egophony is a test to assess breath sounds. It is positive when the patient is asked to pronounce "E" but instead says "A". In bronchophony, the patient is asked to repeat "99" several times in a row. If the words are easily understood and are clear and loud, it indicates an abnormal finding. In pectoriloquy, the patient is asked to whisper "one-two-three." If the whisper is heard clearly and distinctly, it indicates an abnormal finding. Wheezes are heard in asthma when there is bronchoconstriction. Stridor is heard in laryngeal diseases due to the obstruction of the larynx or trachea. p. 468

The nurse expects which corticosteroid may be prescribed for a patient diagnosed with sinusitis? Select all that apply. A. Zileuton B. Clemastine C. Flunisolide D. Triamcinolone E. Fluticasone mometasone

Answers: C. Flunisolide D. Triamcinolone E. Fluticasone mometasone Rationale: Flunisolide, triamcinolone, and fluticasone mometasone are corticosteroids used to treat sinusitis. Zileuton is a leukotriene receptor inhibitor used to treat sinusitis. Clemastine is a first-generation antihistamine used to treat sinusitis. p. 481

Which actions have priority during Ms. Asher's current asthma attack to ensure she is calm and focused on normal breathing? Select all that apply. A. Administer albuterol. B. Provide oral salmeterol. C. Provide a comfortable place for her to sit. D. Identify environmental triggers of the attacks. E. Reassure her that the attack will eventually end.

Answers: C. Provide a comfortable place for her to sit E. Reassure her that the attack will eventually end. Rationale: Finding a comfortable place for Ms. Asher to sit will help her to remain calm. The nurse should reassure Ms. Asher that the attack will end with proper treatment. Providing reassurance will help keep this patient calm.

A patient reports shortness of breath one day after a cholecystectomy. The nurse assesses the right lung sounds and notes dullness to percussion and decreased breath sounds. Which is the most probable reason for the assessment findings? A. Atelectasis B. Pneumonia C. Pneumothorax D. Tension pneumothorax

Rationale: A. Atelectasis Rationale: Atelectasis is a lung condition characterized by collapsed, airless alveoli. There may be decreased or absent breath sounds and dullness to percussion over the affected area. The most common cause of atelectasis is obstruction of the small airways with secretions. This is common in bedridden patients and in postoperative abdominal and chest surgery patients. Pneumonia can have similar findings, but it is highly unlikely to occur one day after surgery. In both pneumothorax and tension pneumothorax, the affected area is hyperresonant. p. 531

The home health nurse provides which instruction for a patient being treated for pneumonia? A. Use a cool mist humidifier to help with breathing. B. Drink at least 4 to 6 glasses of liquids/day. C. Schedule a follow-up chest x-ray in 8 to 10 weeks. D. Expect that it may be several days before the usual sense of well-being returns.

Rationale: A. Use a cool mist humidifier to help with breathing. Rationale: A cool mist humidifier or warm bath may help the patient breathe easier. The nurse should tell the patient to drink plenty of liquids (at least 6 to 10 glasses/day, unless contraindicated). The nurse should explain that a follow-up chest x-ray may be done in 6 to 8 weeks to evaluate the resolution of pneumonia. The nurse should tell patients that it may be several weeks before their usual vigor and sense of well-being return. The older adult or chronically ill patient may have a prolonged period of convalescence. p. 509


संबंधित स्टडी सेट्स

Chapter 8 TCP/IP Internetworking I

View Set

Chapter 2 Statement of Cashflows and Free Cashflow

View Set

Macro Homework Finals Study Guide

View Set